ChaseDream

标题: 41.: GWD-23-Q39 [打印本页]

作者: tonylee123    时间: 2013-10-27 19:59
标题: 41.: GWD-23-Q39
In Gandania, where the government has a monopoly on tobacco sales, the incidence of smoking-related health problems has risen steadily for the last twenty years. The health secretary recently proposed a series of laws aimed at curtailing tobacco use in Gandania. Profits from tobacco sales, however, account for ten percent of Gandania’s annual revenues. Therefore, Gandania cannot afford to institute the proposed laws.



Which of the following, if true, most seriously weakens the argument?


A.All health care in Gandania is government-funded.


B.Implementing the proposed laws is not likely to cause a significant increase in the amount of tobacco Gandania exports.


C.The percentage of revenue Gandania receives from tobacco sales has remained steady in recent years.
D.Profits from tobacco sales far surpass any other single source of revenue for the Gandanian government.


E.No government official in Gandania has ever previously proposed laws aimed at curtailing tobacco use.

答案是A


请问一下这个法案他执行了吗  看到if true 所以是假设他已经执行了 然后cannotafford to institute吗
谢谢好心nn帮忙解困
作者: wwxavier    时间: 2013-10-28 03:07
这个if true是说下面选项的不是说上面信息的。而且题目信息和选项都应看作是已知信息,所以你以后对这个if true完全可以视而不见。




欢迎光临 ChaseDream (https://forum.chasedream.com/) Powered by Discuz! X3.3